Ανισότητα χωρίς να είναι θετικοί

Συντονιστές: achilleas, emouroukos, silouan

ΠΑΠΑΔΟΠΟΥΛΟΣ ΣΤΑΥΡΟΣ
Δημοσιεύσεις: 3600
Εγγραφή: Πέμ Φεβ 27, 2014 9:05 am
Τοποθεσία: ΧΑΛΚΙΔΑ- ΑΘΗΝΑ-ΚΡΗΤΗ

Ανισότητα χωρίς να είναι θετικοί

#1

Μη αναγνωσμένη δημοσίευση από ΠΑΠΑΔΟΠΟΥΛΟΣ ΣΤΑΥΡΟΣ » Κυρ Σεπ 01, 2019 12:41 am

Εστω x,y\in \mathbb{R}

Να δείξετε ότι

(x^{3}+(x+y)^{3})(y^{3}+(x+y)^{3})\leq 4(x+y)^{6}

πότε έχουμε ισότητα;

Διορθώθηκε η εκφώνηση.
Ευχαριστώ τον Σωτήρη Χασάπη ,τον Γιώργο Βισβίκη και JimNt για την επισήμανση.
τελευταία επεξεργασία από ΠΑΠΑΔΟΠΟΥΛΟΣ ΣΤΑΥΡΟΣ σε Κυρ Σεπ 01, 2019 1:30 pm, έχει επεξεργασθεί 1 φορά συνολικά.



Λέξεις Κλειδιά:
Άβαταρ μέλους
JimNt.
Δημοσιεύσεις: 590
Εγγραφή: Παρ Μάιος 20, 2016 3:00 pm

Re: Ανισότητα χωρίς να είναι θετικοί

#2

Μη αναγνωσμένη δημοσίευση από JimNt. » Κυρ Σεπ 01, 2019 11:22 am

ΠΑΠΑΔΟΠΟΥΛΟΣ ΣΤΑΥΡΟΣ έγραψε:
Κυρ Σεπ 01, 2019 12:41 am
Εστω x,y\in \mathbb{R}

Να δείξετε ότι

(x^{3}+(x+y)^{3})(y^{3}+(x+y)^{3})\leq 4(x+y)^{3}

πότε έχουμε ισότητα;
Προφανώς και δεν ισχύει. Θέτω x=ky με y, k θετικό. Πρέπει λοιπόν, y^3(k^3+(k+1)^3)(1+(k+1)^3) \le 4(k+1)^3, που δεν ισχύει για μεγάλα y. (αν θέσουμε k=-1/3 εφαρμόζεται το ίδιο επιχείρημα, καθώς και για αρνητικούς (ίδιο με πάνω αλλά με |y| αρκετά μικρό)).


Bye :')
Άβαταρ μέλους
Demetres
Γενικός Συντονιστής
Δημοσιεύσεις: 8989
Εγγραφή: Δευ Ιαν 19, 2009 5:16 pm
Τοποθεσία: Λεμεσός/Πύλα
Επικοινωνία:

Re: Ανισότητα χωρίς να είναι θετικοί

#3

Μη αναγνωσμένη δημοσίευση από Demetres » Κυρ Σεπ 01, 2019 11:37 am

Μάλλον το δεξί μέλος θέλει έκτη αντί τρίτη δύναμη.

Η ανισότητα είναι προφανής για x=0 ή y=0. Αλλιώς θέτουμε y=kx και το ζητούμενο γίνεται

\displaystyle  4(k+1)^6 \geqslant (k^3+(k+1)^3)(1+(k+1)^3)

Εν αναμονή καλύτερης απόδειξης, αυτό είναι ισοδύναμο (σύμφωνα με το wolframalpha) με το

\displaystyle  (k^2+3k+1)^2(2k^2+3k+2) \geqslant 0

το οποίο ισχύει αφού η διακρίνουσα του 2k^2+3k+2 είναι αρνητική.


ΠΑΠΑΔΟΠΟΥΛΟΣ ΣΤΑΥΡΟΣ
Δημοσιεύσεις: 3600
Εγγραφή: Πέμ Φεβ 27, 2014 9:05 am
Τοποθεσία: ΧΑΛΚΙΔΑ- ΑΘΗΝΑ-ΚΡΗΤΗ

Re: Ανισότητα χωρίς να είναι θετικοί

#4

Μη αναγνωσμένη δημοσίευση από ΠΑΠΑΔΟΠΟΥΛΟΣ ΣΤΑΥΡΟΣ » Κυρ Σεπ 01, 2019 1:32 pm

JimNt. έγραψε:
Κυρ Σεπ 01, 2019 11:22 am
ΠΑΠΑΔΟΠΟΥΛΟΣ ΣΤΑΥΡΟΣ έγραψε:
Κυρ Σεπ 01, 2019 12:41 am
Εστω x,y\in \mathbb{R}

Να δείξετε ότι

(x^{3}+(x+y)^{3})(y^{3}+(x+y)^{3})\leq 4(x+y)^{3}

πότε έχουμε ισότητα;
Προφανώς και δεν ισχύει. Θέτω x=ky με y, k θετικό. Πρέπει λοιπόν, y^3(k^3+(k+1)^3)(1+(k+1)^3) \le 4(k+1)^3, που δεν ισχύει για μεγάλα y. (αν θέσουμε k=-1/3 εφαρμόζεται το ίδιο επιχείρημα, καθώς και για αρνητικούς (ίδιο με πάνω αλλά με |y| αρκετά μικρό)).
Ευχαριστώ.
Πάντως πολύ πιο απλά αν πάρει κάποιος x=y=1 βλέπει ότι δεν ισχύει.

Διόρθωσα την εκφώνηση.


Άβαταρ μέλους
george visvikis
Επιμελητής
Δημοσιεύσεις: 13275
Εγγραφή: Παρ Νοέμ 01, 2013 9:35 am

Re: Ανισότητα χωρίς να είναι θετικοί

#5

Μη αναγνωσμένη δημοσίευση από george visvikis » Κυρ Σεπ 01, 2019 5:23 pm

Θέτω \displaystyle x + y = a,xy = b και καταλήγω στη σχέση:

\displaystyle (2{a^2} - b){({a^2} + b)^2} \ge 0 \Leftrightarrow (2{x^2} + 3xy + 2{y^2}){({x^2} + 3xy + {y^2})^2} \ge 0, που ισχύει. Η ισότητα ισχύει

για 2{x^2} + 3xy + 2{y^2}=0 \Leftrightarrow \boxed{x=y=0} ή \displaystyle {x^2} + 3xy + {y^2} = 0 \Leftrightarrow \boxed{x = \frac{y}{2}\left( { - 3 \pm \sqrt 5 } \right)}


Άβαταρ μέλους
silouan
Επιμελητής
Δημοσιεύσεις: 1398
Εγγραφή: Τρί Ιαν 27, 2009 10:52 pm

Re: Ανισότητα χωρίς να είναι θετικοί

#6

Μη αναγνωσμένη δημοσίευση από silouan » Κυρ Σεπ 01, 2019 6:22 pm

Λίγο διαφορετικά. Λόγω ομοιογένειας, θέτω x+y=1. Τότε
\displaystyle{(x^3+1)(y^3+1)=x^3y^3+x^3+y^3+1=(xy)^3+(x+y)((x+y)^2-3xy))+1 =(xy)^3-3xy+2=(xy-1)^2(xy+2)}.
Προφανώς το μέγιστο ισχύει όταν xy\geq -2. Επιπλέον xy\leq\frac{1}{4} οπότε και ο όρος 1-xy είναι θετικός.
Επομένως εφαρμόζουμε ΑΜ-ΓΜ και έχουμε
\displaystyle{(xy-1)^2(xy+2)=\frac{(1-xy)(1-xy)(2xy+4)}{2}\leq\frac{1}{2}\left(\frac{1-xy+1-xy+2xy+4}{3}\right)^3=4.}

Η ισότητα ισχύει όταν xy=-1 και x+y=1.


Σιλουανός Μπραζιτίκος
ksofsa
Δημοσιεύσεις: 440
Εγγραφή: Κυρ Απρ 18, 2010 9:42 pm

Re: Ανισότητα χωρίς να είναι θετικοί

#7

Μη αναγνωσμένη δημοσίευση από ksofsa » Κυρ Σεπ 01, 2019 6:46 pm

Λίγο διαφορετικά:

Θεωρώ

x+y=1

Τότε

(x^3+1)(y^3+1)-4=x^3y^3+x^3+y^3-3=x^3y^3+x^3+y^3+(-1)^3-2=x^3y^3-3xy-2=

(xy+1)^2(xy-2)\leq 0,

που ισχύει διότι xy\leq \frac{(x+y)^2}{4}=\frac{1}{4}<2


Κώστας
Απάντηση

Επιστροφή σε “Άλγεβρα - Επίπεδο Θαλή/Ευκλείδη (Seniors)”

Μέλη σε σύνδεση

Μέλη σε αυτήν τη Δ. Συζήτηση: Δεν υπάρχουν εγγεγραμμένα μέλη και 0 επισκέπτες